Đến nội dung

Hình ảnh

Cauchy-Schwarz


  • Please log in to reply
Chủ đề này có 176 trả lời

#141
the man

the man

    Thiếu úy

  • Thành viên
  • 589 Bài viết

Cho x,y $\epsilon$ R và 21x^2-36xy+44y^2 $\leqslant$ 27. CMR: x+2y $\geq$ -3

Đặt $x+2y=t \Rightarrow x=t-2y$. Thay vào đầu bài ta có 

 $21(t-2y)^2-36y(t-2y)+44y^2 -27 \leq 0$

Thu gọn và nhóm hợp lí ta được $2(10y-3t)^2+3t^2-27 \leq0$

           $ \Rightarrow 3t^2-27 \leq0 \Rightarrow -3 \leq t \leq3$


"God made the integers, all else is the work of man."

                                                Leopold Kronecker


#142
Ngoc Hung

Ngoc Hung

    Đại úy

  • Điều hành viên THCS
  • 1547 Bài viết

Cho $x+y+z\leq \frac{3}{2}$ và x, y, z > 0. Tìm GTNN của $P=\left ( \frac{1}{x}+\frac{1}{y}+3 \right )\left ( \frac{1}{y}+\frac{1}{z}+3 \right )\left ( \frac{1}{z}+\frac{1}{x}+3 \right )$



#143
the man

the man

    Thiếu úy

  • Thành viên
  • 589 Bài viết

Cho $x+y+z\leq \frac{3}{2}$ và x, y, z > 0. Tìm GTNN của $P=\left ( \frac{1}{x}+\frac{1}{y}+3 \right )\left ( \frac{1}{y}+\frac{1}{z}+3 \right )\left ( \frac{1}{z}+\frac{1}{x}+3 \right )$

Đặt $\frac{1}{x}+\frac{1}{y}=a,\frac{1}{y}+\frac{1}{z}=b,\frac{1}{x}+\frac{1}{z}=c$

  $\Rightarrow P=(a+3)(b+3)(c+3)=abc+9(a+b+c)+3(ab+bc+ca)+27$

  $\geq abc+27+27\sqrt[3]{abc}+9\sqrt[3]{a^2b^2c^2}$

$xyz\leq \left ( \frac{x+y+z}{3} \right )^3=\frac{1}{8}\Rightarrow abc=\left ( \frac{1}{x}+\frac{1}{y} \right )\left( \frac{1}{y}+\frac{1}{z} \right )\left( \frac{1}{x}+\frac{1}{z} \right ) \geq \frac{8}{xyz}\geq 64$

$\Rightarrow P\geq 64+27+144+108=343$


"God made the integers, all else is the work of man."

                                                Leopold Kronecker


#144
lamdaika

lamdaika

    Binh nhất

  • Thành viên
  • 32 Bài viết
2, Cho a, b, c > 0 thỏa mãn: $ab + bc + ca = 3$. Chứng minh rằng:
 
$\frac{1}{1+a^{^{2}}+b^{2}}+\frac{1}{1+a^{^{2}}+b^{2}}+\frac{1}{1+a^{^{2}}+b^{2}}\leq 1 $

Bài viết đã được chỉnh sửa nội dung bởi votruc: 28-07-2015 - 18:43


#145
Air Force

Air Force

    Trung sĩ

  • Thành viên
  • 145 Bài viết

- Như chúng ta đã biết, bất đẳng thức Cauchy-Schwarz có dạng như sau:
Với hai dãy số thực $(a_{1}, a_{2}, ..., a_{m})$ và $(b_{1}, b_{2}, ..., b_{m})$ ta luôn có bất đẳng thức sau:
$(a_{1}^2+a_{2}^2+...+ a_{m}^2)(b_{1}^2+b_{2}^2+...+b_{m}^2) \geq (a_{1}b_{1}+a_{2}b_{2}+...+a_{m}b_{m})^2$
Dấu đẳng thức xảy ra khi và chỉ khi $\dfrac{a_{1}}{b_{1}}= \dfrac{a_{2}}{b_{2}}=...= \dfrac{a_{m}}{b_{m}}$
- Nó cũng có một số hệ quả:
1, Bất đẳng thức Schwarz:
Với hai dãy số thực $(a_{1}, a_{2}, ..., a_{m})$ và $(b_{1}, b_{2}, ..., b_{m})$ sao cho $b_{i} \geq 0$ ta luôn có bất đẳng thức:
$\dfrac{a_{1}^2}{b_{1}}+ \dfrac{a_{2}^2}{b_{2}}+...+ \dfrac{a_{m}^2}{b_{m}} \geq \dfrac{(a_{1}+a_{2}+...+a_{m})^2}{b_{1}+b_{2}+...+b_{m}}$
2, Bất đẳng thức Minkovsky:
Với 2 dãy số thực $\Large (a_{1}, a_{2}, ..., a_{m})$ và $\Large (b_{1}, b_{2}, ..., b_{m})$ ta có:
$\Large \sum\limits_{i=1}^{m} \sqrt{a_{i}^2+b_{i}^2} \geq \sqrt{(\sum\limits_{i=1}^{m} a_{i})^2+(\sum\limits_{i=1}^{m} b_{i})^2}$
3, Với mọi dãy số thực $\Large (a_{1}, a_{2}, ..., a_{m})$ ta có:
$\Large (a_{1}^2+a_{2}^2+...+a_{m})^2 \leq n(a_{1}^2+a_{2}^2+...+a_{m}^2)$
- Đây là một bất đẳng thức rất thông dụng với các bạn THCS và hay được dùng trong các kì thi. 
Sau đây là một số bài tập ứng dụng:
1)Cho $|x|<1$ và $|y|<1$. CMR:
$\dfrac{1}{1-x^2}+ \dfrac{1}{1-y^2} \geq \dfrac{2}{1-xy}$
2)CM bất đẳng thức sau với $x$ là số thực không âm:
$\dfrac{2 \sqrt{2}}{\sqrt{x+1}}+ \sqrt{x} \leq \sqrt{x+9}$
3) $a,b,c >0$. CMR: $abc(a+b+c) \leq a^3b+b^3c+c^3a$
4)CMR:
$\sqrt{abc}+ \sqrt{(1-a)(1-b)(1-c)} <1 $
với mọi $a,b,c \in (0;1)$
5)Tìm min:
$\sum \limits_{i=1}^{n} (x_{i}+ \dfrac{1}{x_{i}})^2$
với $\sum \limits_{i=1}^{n} x_{i}=1$    



#146
phihungtf

phihungtf

    Lính mới

  • Thành viên mới
  • 5 Bài viết

Đề:

 

Chứng minh rằng:

          

             $\sqrt{1} + \sqrt{2} + \sqrt{3} +...+ \sqrt{n} \leqslant n\sqrt{\frac{n+1}{2}}$

 

Giúp em giải nhanh trong ngày hôm nay nhé! <3



#147
yeutoan2001

yeutoan2001

    Thượng sĩ

  • Thành viên
  • 231 Bài viết

 

2, Cho a, b, c > 0 thỏa mãn: $ab + bc + ca = 3$. Chứng minh rằng:
 
$\frac{1}{1+a^{^{2}}+b^{2}}+\frac{1}{1+a^{^{2}}+b^{2}}+\frac{1}{1+a^{^{2}}+b^{2}}\leq 1 $

 

$\sum \frac{c^{2}+1+1}{(a^{2}+b^{2}+1)(1+1+c^{2})}\leq \sum \frac{c^{2}+2}{(a+b+c)^{2}}=\frac{a^{2}+b^{2}+c^{2}+6}{(a+b+c)^{2}}=\frac{a^{2}+b^{2}+c^{2}+6}{a^{2}+b^{2}+c^{2}+6}=1$



#148
yeutoan2001

yeutoan2001

    Thượng sĩ

  • Thành viên
  • 231 Bài viết

Đề:

 

Chứng minh rằng:

          

             $\sqrt{1} + \sqrt{2} + \sqrt{3} +...+ \sqrt{n} \leqslant n\sqrt{\frac{n+1}{2}}$

 

Giúp em giải nhanh trong ngày hôm nay nhé! <3

Bunhia cho n số bạn nhé:

$(\sqrt{1}.1+\sqrt{2}.1+...+\sqrt{n}.1)^{2}\leq (1+1+...+1)(1+2+...+n)=n.\frac{n(n+1)}{2}$

Căn bậc hai hai vế ta có đpcm nhé



#149
phihungtf

phihungtf

    Lính mới

  • Thành viên mới
  • 5 Bài viết

Bunhia cho n số bạn nhé:

$(\sqrt{1}.1+\sqrt{2}.1+...+\sqrt{n}.1)^{2}\leq (1+1+...+1)(1+2+...+n)=n.\frac{n(n+1)}{2}$

Căn bậc hai hai vế ta có đpcm nhé

cám ơn bạn nhiều! <3



#150
yeutoan2001

yeutoan2001

    Thượng sĩ

  • Thành viên
  • 231 Bài viết

cám ơn bạn nhiều! <3

Xin like 



#151
harryhuyen

harryhuyen

    Hạ sĩ

  • Thành viên
  • 63 Bài viết

Ờ để anh giải thử:
$\large\ a^2+1 \geq 2a;b^2+1 \geq 2b;c^2+1 \geq 2c $
Ta CM $\large\ VT \leq \dfrac{1}{2} $ (Nhẩm $a=b=c=1$)
$\Leftrightarrow \large\sum\dfrac{a}{a+b+1} \leq 1 $
$\Leftrightarrow \large\sum\dfrac{b+1}{a+b+1} \geq 2 $
Ta có $\large\sum\dfrac{b+1}{a+b+1} \geq \dfrac{(a+b+c+3)^2}{\sum[(b+1)(a+b+1)} \geq 2 $ (Biến đổi tương đương hoặc khai triển vế ở mẫu đưa về $\leq \dfrac{1}{2}(a+b+c+3)^2 $ image004.gif)??? hình như bị ngược dấu



#152
Nguyenphuctang

Nguyenphuctang

    Sĩ quan

  • Banned
  • 499 Bài viết

Cho a, b, c > 0; $\sum a^{2}=1.$Chứng minh rằng:

$\sum \frac{a^{2}+1}{b^{2}+ac}\geq 6$

 

Nguồn: Sáng tác.


Bài viết đã được chỉnh sửa nội dung bởi Nguyenphuctang: 15-11-2016 - 15:03


#153
harryhuyen

harryhuyen

    Hạ sĩ

  • Thành viên
  • 63 Bài viết

cho cac so thuc a,b,c,d thoa man $(a^2+1)(b^2+1)(c^2+1)(d^2+1)=16 $

cm   $ -3\leq ab+ac+ad+bc+bd+cd-abcd\leq 5 $


Bài viết đã được chỉnh sửa nội dung bởi harryhuyen: 15-11-2016 - 17:20


#154
harryhuyen

harryhuyen

    Hạ sĩ

  • Thành viên
  • 63 Bài viết

mọi người giúp em với cho a,b,c,d là các số thực dương thỏa     $  (a^2+1)(b^2+1)(c^2+1)(d^2+1)=16 $

cm $-3\leq ab+ac+ad+bc+bd+cd-abcd\leq 5$



#155
hieumetoan

hieumetoan

    Binh nhì

  • Thành viên mới
  • 19 Bài viết

Vậy là box này thành lập dc gần 10 rồi các anh chị ơi.Chúng ta chuẩn bị kỉ niệm nhé :icon6:  :icon6:  :icon6:


1+1=2


#156
yeutoan2001

yeutoan2001

    Thượng sĩ

  • Thành viên
  • 231 Bài viết

mọi người giúp em với cho a,b,c,d là các số thực dương thỏa     $  (a^2+1)(b^2+1)(c^2+1)(d^2+1)=16 $

cm $-3\leq ab+ac+ad+bc+bd+cd-abcd\leq 5$

 Có: $  (a^2+1)(b^2+1)(c^2+1)(d^2+1)=16 $ 

 <=> $16=(a^{2}b^{2}+a^{2}+b^{2}+1)(c^{2}+d^{2}+c^{2}d^{2}+1)$

<=> $16=((ab-1)^{2}+(a+b)^{2})((cd-1)^{2}+(c+d)^{2})\geq ((ab-1)(cd-1)+(a+b)(c+d))^{2}$

Từ đây dễ dàng =>  ĐPCM 



#157
Thuat ngu

Thuat ngu

    Trung sĩ

  • Thành viên
  • 139 Bài viết

Bài này cũng mang dáng dấp của Cauchy-Schwarz, mời mọi người giải: 

Cho a,b,c >0 thỏa mãn a+b+c=3. CM: $\frac{a}{b^{2}\left ( ca+1 \right )}+\frac{b}{c^{2}\left ( ab+1 \right )}+\frac{c}{a^{2}\left ( bc+1 \right )}\geq \frac{9}{\left ( 1+abc \right )\left ( ab+bc+ca \right )}$



#158
Ren

Ren

    Hạ sĩ

  • Thành viên
  • 67 Bài viết

Cho $x+y+z\leq \frac{3}{2}$ và x, y, z > 0. Tìm GTNN của $P=\left ( \frac{1}{x}+\frac{1}{y}+3 \right )\left ( \frac{1}{y}+\frac{1}{z}+3 \right )\left ( \frac{1}{z}+\frac{1}{x}+3 \right )$

\[P = \left( {{{\sqrt[3]{{\frac{1}{x}}}}^3} + {{\sqrt[3]{{\frac{1}{y}}}}^3} + {{\sqrt[3]{3}}^3}} \right)\left( {{{\sqrt[3]{{\frac{1}{y}}}}^3} + {{\sqrt[3]{{\frac{1}{z}}}}^3} + {{\sqrt[3]{3}}^3}} \right)\left( {{{\sqrt[3]{{\frac{1}{z}}}}^3} + {{\sqrt[3]{{\frac{1}{x}}}}^3} + {{\sqrt[3]{3}}^3}} \right) \ge {\left( {\sqrt[3]{{\frac{1}{{xyz}}}} + \sqrt[3]{{\frac{1}{{xyz}}}} + 3} \right)^3}(chebyshev)\]
\[ Mà : \frac{{27}}{8} \ge x + y + z \ge 3\sqrt[3]{{xyz}} <  =  > xyz \le \frac{1}{8}\]

\[ =  > P \ge {\left( {\sqrt[3]{{\frac{1}{{xyz}}}} + \sqrt[3]{{\frac{1}{{xyz}}}} + 3} \right)^3} \ge {\left( 7 \right)^3}\]

\[ Dấu "="  <  =  > a = b = c = \frac{1}{2}\]


Bài viết đã được chỉnh sửa nội dung bởi Ren: 14-02-2017 - 13:53


#159
viet9a14124869

viet9a14124869

    Trung úy

  • Thành viên
  • 903 Bài viết

 Có: $  (a^2+1)(b^2+1)(c^2+1)(d^2+1)=16 $ 

 <=> $16=(a^{2}b^{2}+a^{2}+b^{2}+1)(c^{2}+d^{2}+c^{2}d^{2}+1)$

<=> $16=((ab-1)^{2}+(a+b)^{2})((cd-1)^{2}+(c+d)^{2})\geq ((ab-1)(cd-1)+(a+b)(c+d))^{2}$

Từ đây dễ dàng =>  ĐPCM 

mình xin mách nhỏ rằng chỗ này nên là dấu trừ thì ta sẽ có q.e.d


                                                                    SÓNG BẮT ĐẦU TỪ GIÓ

                                                                    GIÓ BẮT ĐẦU TỪ ĐÂU ?

                                                                    ANH CŨNG KHÔNG BIẾT NỮA 

                                                                    KHI NÀO...? TA YÊU NHAU .


#160
viet9a14124869

viet9a14124869

    Trung úy

  • Thành viên
  • 903 Bài viết

Cho a, b, c > 0; $\sum a^{2}=1.$Chứng minh rằng:

$\sum \frac{a^{2}+1}{b^{2}+ac}\geq 6$

 

Nguồn: Sáng tác.

Ta có $\sum \frac{a^2}{b^2+ac}\geq \frac{(a+b+c)^2}{a^2+b^2+c^2+ab+bc+ca}$

Và $\sum \frac{1}{b^2+ac}\geq \frac{9}{a^2+b^2+c^2+ab+bc+ca}$

$\Rightarrow \sum \frac{a^2+1}{a^2+b^2+c^2+ab+bc+ca}\geq \frac{(a+b+c)^2+9}{a^2+b^2+c^2+ab+bc+ca}=\frac{10+2ab+2bc+2ca}{1+ab+bc+ca}$

Đặt ab+bc+ca=x $\Rightarrow x\leq 1$

Thay vào đẳng thức ta có q.e.d


                                                                    SÓNG BẮT ĐẦU TỪ GIÓ

                                                                    GIÓ BẮT ĐẦU TỪ ĐÂU ?

                                                                    ANH CŨNG KHÔNG BIẾT NỮA 

                                                                    KHI NÀO...? TA YÊU NHAU .





0 người đang xem chủ đề

0 thành viên, 0 khách, 0 thành viên ẩn danh